Answer with explaination every steps​

Answer With Explaination Every Steps

Answers

Answer 1

Answer:

y = - 35

Step-by-step explanation:

Expressing as a fraction , that is

[tex]\frac{y}{-7}[/tex] = 5 ( multiply both sides by - 7 to clear the fraction )

y = - 35


Related Questions

Which are correct Representation of the any quality 6x>_3+4(2x-1)?select three options

Answers

Answer:

1) 1 ≥ 2·x

2) 6·x ≥ 3 + 8·x - 4

3) 1/2 ≥ x The third option and the first number line inequality diagram, please see attached drawing of the number line created with MS Visio

Step-by-step explanation:

The given inequality is presented as follows;

6·x ≥ 3 + 4·(2·x - 1)

By expanding the right hand side of the inequality, we get;

6·x ≥ 3 + 8·x - 4

4 - 3 = 1 ≥ 8·x - 6·x = 2·x

∴ 1 ≥ 2·x

1/2 ≥ x

Therefore, the correct options are;

1) 1 ≥ 2·x

2) 6·x ≥ 3 + 8·x - 4

3) 1/2 ≥ x The third option and the first number line inequality diagram

A condition statement is logically equivalent to a biconditional statement. true or false​

Answers

Answer:

true

Step-by-step explanation:

Because a logically equivalent is the same as biconditional statement

Step-by-step explanation:

hello the answer is true, you can check but it's obviously true

Factorise 2ab+2ac. Please

Answers

Answer:

2a(b+c)

Step-by-step explanation:

Find HCF (2a)

Then factorise

Answer:

2a(b+c)

.............

What's 70 x4 I just wanna see if it’s really going to help me

Answers

Step-by-step explanation:

70

*4

___

280.......ans

In right ΔDEF, DF = 20, m∠ F = 90˚, EF = 17. Which of the following is true? Select all that apply

Answers

Answer:

option 1

Step-by-step explanation:

I am not sure, what your are asking about.

in your text you define the length of DF (20), but the answer options (only 2 visible) also specify DF but differently.

so ... ?

when you said DF=20, did you actually mean DE ?

under that assumption

DE = 20 = Hypotenuse of the right-angled triangle (the opposite side of the 90 degree angle).

EF = 17

based on Pythagoras

c² = a² + b²

we have here now

20² = 17² + DF²

400 = 289 + DF²

DF² = 400 - 289 = 111

DF = sqrt(111)

the red line below is perpendicular to which of the following

Answers

The red line is perpendicular to the y-axis.

An x-method chart shows the product a c at the top of x and b at the bottom of x. Below the chart is the expression a x squared + b x + c. What are the factors of x2 – 144? And

Answers

Answer:

[tex](x -12)(x + 12)[/tex]

Step-by-step explanation:

Given

See attachment for chart

Required

The factors of [tex]x^2 - 144[/tex]

First, express [tex]x^2 - 144[/tex] as [tex]ax^2 + bx + c[/tex]

So, we have:

[tex]x^2 - 144 = x^2 + 0x - 144[/tex]

Compare the above expression to: [tex]ax^2 + bx + c[/tex]

We have:

[tex]ax^2 + bx + c = x^2 + 0x - 144[/tex]

So:

[tex]a =1[/tex]

[tex]b =0[/tex]

[tex]c = -144[/tex]

and

[tex]a * c = d * e[/tex]

Calculate ac

[tex]a* c = 1 * -144[/tex]

[tex]a* c = -144[/tex]

Rewrite as:

[tex]a* c = -12 * 12[/tex]

Recall that:

[tex]a * c = d * e[/tex]

Hence:

[tex]d = -12; e = 12[/tex]

So, on the x chart, we have:

        ac

d                  e

        b

This gives:

        -144

-12                  12

        0

The factors are

[tex](x + d)(x + e)[/tex]

[tex](x -12)(x + 12)[/tex]

Answer:

✔ (x – 12)

 and  

✔ (x + 12)

Step-by-step explanation:

9. Determine the volume of concrete needed to build a ramp in the shape of a triangular prism to the nearest tenth of a cubic metre. ​

Answers

Answer:

67.5

Step-by-step explanation:

I've learned this multiple ways but in my opinion this is the easiest. Just times everything together so, 40 x 2.5 x 1.2 which equals 135 and then divide 135 by 2 which equals your answer of 67.5 which is rounded to the nearestt tenth already.

How can I get the answer

Answers

Answer:

A.

Step-by-step explanation:

To find the inverse of a function, first make f(x) a y

So,

y = [tex]\frac{12}{x}[/tex] -18

Then switch the x and the y

x = [tex]\frac{12}{y}[/tex] - 18

Solve, for y

SOMEONE PLEASE HELP ME OUT!!!!

Answers

Answer:

40/30

Step-by-step explanation:

Since tan∅= o/a, 40 is opposite, and 30 is adjacent to angle A, 40/30 is the ratio for tanA.

Answer:40/30

Step-by-step explanation:

....

Look at the graph below. What type of function is represented by this graph?

Someone come through with the answers pls

Answers

So you need to get your slope formed then rise over run and calculate your graph. Y=Mx+b

jayce travels 30 miles per hour in her car.how many miles does she travel in 4 hours

Answers

Answer:

120 miles

Step-by-step explanation:

30 miles per hour * 4 hours

120 miles

Answer:

She travels 120 miles in 4 hours.

Step-by-step explanation:

She travel in 4 hours = 30 miles × 4 = 120 miles

She travels 120 miles in 4 hours

calculate the exact value of 1 1/3- 3 5/6+ 5 1/9​

Answers

[tex]\displaystyle\bf 1\frac{1}{3} -3\frac{5}{6} +5\frac{1}{9} =5+1-3+\frac{1^{/6}}{3} +\frac{1^{/2}}{9} -\frac{5^{/3}}{6}\\\\\\\ =3+\frac{6+2-15}{18} =3-\frac{7}{18}=\boxed{2\frac{11}{18} }[/tex]

on a coordinate gride what is the distance between (1,3) and (6,15)

Answers

Answer:

13

Step-by-step explanation:

Distance between points (1, 3) and (6, 15) is 13

13 is the right answer for sure

divide 3 divided by 2/5

Answers

Answer:

[tex]{ \tt{ = 3 \div \frac{2}{5} }} \\ = { \tt{3 \times \frac{5}{2} }} \\ = \frac{15}{2} [/tex]

Show that x +1 is a factor of f(x) =2x^3 +3x^2 - 5x - 6

Answers

Answer:

Step-by-step explanation:

To solve this, we can use polynomial long division.

Seeing the picture, we first divide x (the variable to the largest degree in x+1) from 2x³. We divide from 2x³ because that is the variable with the largest degree in the polynomial. That is equal to 2x², so we put that on top and subtract (x+1) * (2x²) from the polynomial. Then, we repeat the process, but with x² instead of 2x³, and again with -6x as the variable with the largest degree.

What is the slope of the graph shown below

Answers

Answer:

B=-5

Step-by-step explanation:

Slope=rise/run

The line passes in

P1(-1,3)

and

P2(0,-2)

So slope=(3-(-2))/(-1-0)=5/-1=-5

algebra help pls answer

Answers

Answer:

[tex]\frac{-1+1}{0-(-2)}=0[/tex]

[tex]ANSWER: A)~0[/tex]

------------------------------

HOPE IT HELPS

HAVE A GREAT DAY!!

A bag contains 6 black tiles, 5 white tiles, and 4 blue tiles. Event A is defined as drawing a white tile from the bag on the first draw, and event B is defined as drawing a black tile on the second draw. If two tiles are drawn from the bag, one after the other without replacement, what is P(A and B) expressed in simplest form? A. 4/45 B. 1/7 C. 4/15 D. 5/14

Answers

Answer:

5/14

Step-by-step explanation:

There are 15 tiles in total

5 white| 6 black | 4 blue

event A results in the subject pulling a white tile and not replacing it

5-1= 4

so the first answer should be 4/15

event B results in the subject pulling another tile, a black one and not replacing it.

6-1= 5

given this answer, there is one less tile in the total, since we removed another tile.

So our answer would be-

5/14 or D

(x2/5)n

For what value of n, written as a decimal, will the expression equal x?

Answers

Given:

The expression is:

[tex]\left(x^{\frac{2}{5}\right)^n[/tex]

To find:

The decimal value of n so that the value of the given expression is equal to x.

Solution:

We have,

[tex]\left(x^{\frac{2}{5}\right)^n[/tex]

This expression is equal to x.

[tex]\left(x^{\frac{2}{5}\right)^n=x[/tex]

[tex]x^{\frac{2}{5}n}=x^1[/tex]                     [tex][\because (a^m)^n=a^{mn}][/tex]

On comparing the exponents, we get

[tex]\dfrac{2}{5}n=1[/tex]

[tex]2n=5[/tex]

[tex]n=\dfrac{5}{2}[/tex]

[tex]n=2.5[/tex]

Therefore, the required value of n is 2.5.

Here is the distribution of blood types from a group
of randomly selected people:
o
А
B
AB
Blood
Type:
Probability:
0.49
0.27
0.20
?
What is the probability of type AB blood?

Answers

0.04 is the probability of type AB blood

I need the answer ASAP anyone could help me please

Answers

Answer:

Is it the answer is C?

2+4+3+5+1=15

You want to add each persons total marbles together. So you have… 2 (Ray) + 4 (Ann) + 3 (Roy) + 5 (Zoie) + 1 (Raj) = 15 (total)

Answer: (C) 15

Pls help ASAP!!!!!

Find the average rate of change from d=4
to d=11 for the function f(d) = 5(1.02)^d. Describe the process and steps he used and explain what the average rate of change represents.

Answers

Answer:

0. 116

Step-by-step explanation:

The function is given as :-

[tex]\boxed{f(d) = 5(1.02)^d }[/tex]

and we have to find the rate of change from d = 4 to d = 11

[tex]\boxed{\blue{\mathfrak{Rate\: of\: change = \frac{final\:output-intital\:output}{final\:input-initial\:input} } }}[/tex]

The final input value is 11 whereas the initial input value is 4.

The final and initial outputs can be calculated by placing the respective values of initial and final inputs (that are 4 and 11).

[tex]{\underline{Initial\:Output}}[/tex]

f(4) = [tex]5(1.02)^4[/tex]

f(4) = 5 × 1. 08

f(4) = 5. 41

[tex]{\underline{Final\:Output}}[/tex]

f(11) = [tex]5(1.02)^11[/tex]

f(11) = 5 × 1. 24

f(11) = 6. 22

[tex]\underline{Avg\:Rate \: of \: change} = \frac{6. 22-5.41}{11-4} \\ = \frac{0.81}{7} \\ = 0.116 [/tex]

[tex]\bigstar[/tex] Hence, the average rate of change is [tex]\red{\underline{\pmb{0. 116}}}[/tex]

The triangles are similar.

What is the value of x?

Enter your answer in the box.

Answers

Answer:

x = 23

Step-by-step explanation:

x = 39 divided by 3

x = 13

but it’s saying that x - 10 meaning 13 - 10 which can’t be the answer so we must add 10 to 13 which equals 23 and that’s our final answer.

Final Answer: 23

add: -38+6+27+(-8)+126

Answers

Answer:

113

Step-by-step explanation:

Which point is in the solution set of this system inequalities?

A. (0,0)

B. None of these

C. (5,1)

D. (3,7)

Answers

Answer:

B

Step-by-step explanation:

To find which ordered pairs are solutions to the inequalities we can simply plug in the x and y values of the ordered pairs into the inequalities and if the equation is true for both inequalities then the ordered pair is a solution to the inequalities.

For (0,0)

x = 0

y = 0

y > x + 5

Substitute 0 for y and x

0 > 0 + 5

Simplify right side

0 > 5

The inequality is not true as 5 is greater than 0, not less than. So immediately we can eliminate answer choice A.

For (5,1).

x = 5

y = 1

y > x + 5

Substitute 5 for x and 1 for y

1 > 5 + 5

Simplify right side

1 > 10

Again, the equation is not true as 1 is not greater than 10. This means that c cannot be the answer

For (3,7)

x = 3

y = 7

y > x + 5

Substitute 3 for x and y for 7

7 > 3 + 5

Simplify right side

7 > 8

7 is not greater than 8 meaning that (3,7) cannot be a solution to the inequalities

None of the ordered pairs created true equations hence the answer is B


If f(x) is an exponential function where f(-1.5)=26 and f(5.5)=7 then find the value of f(10), to the nearest hundredth

Answers

Answer:

5.0956537e-46

Step-by-step explanation:

^ means root

* means multiply

f(x) = ab^x

f(-1.5)=26 = ab^-1.5

f(5.5)=7 = ab^5.5

f(5.5)/f(-1.5) = 26/7 = ab^5.5/ab^-1.5

3.714 = b^7

b = 3.714^-7

f(-1.5)=26 = ab^-1.5

26 = ab^-1.5

26 = a * 3.714^-7

a = 3.714^-7 / 26

a = 0.00000394578

f(10) = ab^10

f(10) = 0.00000394578 * (3.714^-7)^10

f(10) = 5.0956537e-46

Tickets to a football final are selling well. On Thursday, 47 of the tickets are sold. On Friday, 14 of the tickets are sold. What fraction of tickets are available to sell on Saturday?

Answers

The question seems incomplete ; as the total number of tickets to be sold isn't given.

Answer:

61 / X

Step-by-step explanation:

Let's take the total Number of tickets to be sold as : X

Number of tickets sold on Thursday = 47

Number sold on Friday = 14

Fraction of tickets available for sale on Saturday :

(Total number of tickets already sold) / Total number of tickets to be sold

(Thursday + Friday sales) / total number of tickets to be sold

Fraction available for sale on Saturday = (47+14) / X

Fraction available for sale on Saturday = 61 / X

Kindly put value of x = total number of tickets available for sale to get the exact fraction.

Find the area of the triangle bounded by the lines y=x y=-x and y=6.

Answers

S=6*12=72

High of triangle is 6

The area of the triangle bounded by the lines y=x y=-x and y=6 is 36 units.

What is area of triangle?

The formula for finding area could be represented in the form of determinants as given below.

[tex]A = \frac{1}{2} \left[\begin{array}{ccc}x1&y1&1\\x2&y2&1\\x3&y3&1\end{array}\right][/tex]

First, we need to find the coordinates of the point of intersection of these lines.

y = x

y = -x

Adding the two equations,

2y = 0

y = 0

x = 0

coordinate: (0, 0)

y = x

y = 6

Subtracting the two equations,

0 = x - 6

x = 6

coordinate: (6, 6)

y = -x

y = 6

Subtracting the two equations,

- x - 6 = 0

x = -6

coordinate: (-6, 6)

Calculating area of triangle bounded by the given line:

Area of triangle =

[tex]\frac{1}{2}\left[\begin{array}{ccc}0&0&1\\6&6&1\\-6&6&1\end{array}\right][/tex]  =  [tex]\frac{1}{2} (36 + 36) = \frac{72}{2} = 36[/tex]

Learn more about area of triangle here

https://brainly.com/question/19305981

#SPJ2

Use the drop-down menus to describe the key aspects of the function
f(x) = –x2 – 2x – 1.

Answers

Answer:

Step-by-step explanation:

Given function is,

f(x) = -x² - 2x - 1

     = -(x² + 2x + 1)

     = -(x + 2)²

Comparing this equation with the vertex form of a quadratic function,

f(x) = a(x - h)² + k

Here, (h, k) is the vertex.

Vertex of the function is (-2, 0)

Leading coefficient of the function = -1

Therefore, parabola will open downwards.

Function will be increasing in the interval (-∞, -2).

Function will be decreasing in the interval (-2, -∞).

Domain of the function → (-∞, ∞)

Range of the function → (-∞, 0]

Answer:

Step-by-step explanation:

edge

Other Questions
Which of the following is an example of an overly broad generalization? (RIGHT ANSWER GETS BRAINLIEST) A) Either you become a physicist, or you enjoy reading great literature.B) Everyone who voted for Governor Jones does not care about education reform.C) Jack had never met a person from Montana before, so his new friend must not be from Montana.D) Calculus is a difficult subject, so I will buy all of my biology textbooks today. help leee plzzzzzzzbbbnbbbb A court may pierce an LLC's veil if a.members treat the LLC like a separate organization. b.members keep their assests and the assets of the LLC separate. c.the LLC has too many members. d.members fail to provide adequate capital. For example, in the Introduction to Bulfinchs Mythology, Bulfinch writes:The Greek poets believed the earth to be flat and circular, their own country occupying the middle of it, the central point being either Mount Olympus, the abode of the gods, or Delphi, so famous for its oracle.What can be inferred by this information?The Greeks didnt travel far beyond their own region.Ancient Greeks at the center of the mapThe poets did not respect the gods as did the average person.The gods were not all knowing. please answer in 3 or 2 sentences. Ill try to give brainliest to whoever can answer in your best way.1. How would a British NEWS reporter DESCRIBE the Boston massacre?2. How would a American NEWS reporter DESCRIBE the Boston massacre? Find the other endpoint of the line segment with the given endpoint and midpoint. Inearest tenth.Endpoint (-8, 8) Midpoint (5,-3) The doctor has ordered Claforan 1 g in 100 ml D5W to run IV piggyback for 30 minutes twice daily. The pharmacy sends Claforn 2 g in a powdered form, which when reconstituted has a concentration of 180 mg Claforan per ml. How much Claforn will you add to the bag of D5W Does anyone know how to solve these? In order to solve a quadratic equation by completing the square, what does the coefficient of the squared term need to be? If the coefficient is not equal to this, what does your first step need to be to complete the sqaure? At room temperature, Mercury exist in the liquid state true or false What is the HCPCS Level II code for the compounded concentrated form of 0.5 mg Levalbuterol HCL when inhaled True or false, If an atom is charged negative, it contains more electrons than protons. The mean of a data set is observed to be very different from its median, representing a strong skewness. However, the 1.5 IQR rule reveals that there are no outliers. Which of the following is correct, if the sample size is 100?a. A normal quantile plot of the data follows a diagonal line, and the t-procedure is appropriate to use. b. A normal quantile plot of the data does not follow a diagonal line, and the t- procedure is not appropriate to use. c. A normal quantile plot of the data follows a diagonal line, and the t-procedure is not appropriate to use. d. A normal quantile plot of the data does not follow a diagonal line, and the t- procedure is appropriate to use. Whats a change agent here Egyptian Culture and Art: TutorialQuestion 2What does the Great Sphinx represent? 30 POINTS !!! what is the total surface area of the square pyramid? 5in height abs 8in base 2040104144 help pls, stuck on this 18x-108-----------6x-54simplify the expressionA. B. C. D. Calculate the vapor pressure (in torr) at 298 K in a solution prepared by dissolving 46.8 g of the non-volatile non-electrolye glucose in 117 g of methanol. The vapor pressure of methanol at 298 K is 122.7 torr. Enter your answer to 2 decimal places. What is the next step after gathering information when making smart food choices?